Difference between revisions of "2003 AIME I Problems/Problem 7"
m (img) |
(asymptote) |
||
Line 3: | Line 3: | ||
== Solution == | == Solution == | ||
− | + | <center><asy> | |
+ | size(220); | ||
+ | pointpen = black; pathpen = black + linewidth(0.7); | ||
+ | pair O=(0,0),A=(-15,0),B=(-6,0),C=(15,0),D=(0,8); | ||
+ | D(D(MP("A",A))--D(MP("C",C))--D(MP("D",D,NE))--cycle); | ||
+ | D(D(MP("B",B))--D); D((0,-4)--(0,12),linetype("4 4")+linewidth(0.7)); | ||
+ | MP("6",B/2); MP("15",C/2); MP("9",(A+B)/2); | ||
+ | </asy></center> <!-- Asymptote replacement for Image:2003_I_AIME-7.png by azjps --> | ||
− | Denote the height of <math>\triangle ACD</math> as <math>h</math>, <math>x = AD = CD</math>, and <math>y = BD</math>. Using the Pythagorean theorem, we find that <math>h^2 = y^2 - 6^2</math> and <math>h^2 = x^2 - 15^2</math>. Thus, <math>y^2 - 36 = x^2 - 225 \Longrightarrow x^2 - y^2 = 189</math>. The LHS is [[difference of squares]], so <math>(x + y)(x - y) = 189</math>. As both <math>x,\ y</math> are integers, <math>x+y,\ x-y</math> must be integral divisors of 189. | + | Denote the height of <math>\triangle ACD</math> as <math>h</math>, <math>x = AD = CD</math>, and <math>y = BD</math>. Using the [[Pythagorean theorem]], we find that <math>h^2 = y^2 - 6^2</math> and <math>h^2 = x^2 - 15^2</math>. Thus, <math>y^2 - 36 = x^2 - 225 \Longrightarrow x^2 - y^2 = 189</math>. The LHS is [[difference of squares]], so <math>(x + y)(x - y) = 189</math>. As both <math>x,\ y</math> are integers, <math>x+y,\ x-y</math> must be integral divisors of <math>189</math>. |
− | The divisors of 189 are <math>(1,189)\ (3,63)\ (7,27)\ (9,21)</math>. This yields the four potential sets for <math>(x,y)</math> as <math>(95,94)\ (33,30)\ (17,10)\ (15,6)</math>. The last is not a possibility since it simply [[degenerate]]s into a [[line]]. The sum of the three possible perimeters of <math> | + | The pairs of divisors of <math>189</math> are <math>(1,189)\ (3,63)\ (7,27)\ (9,21)</math>. This yields the four potential sets for <math>(x,y)</math> as <math>(95,94)\ (33,30)\ (17,10)\ (15,6)</math>. The last is not a possibility since it simply [[degenerate]]s into a [[line]]. The sum of the three possible perimeters of <math> |
− | \triangle ACD</math> is equal to <math>3(AC) + 2(x_1 + x_2 + x_3) = 90 + 2(95 + 33 + 17) = 380</math>. | + | \triangle ACD</math> is equal to <math>3(AC) + 2(x_1 + x_2 + x_3) = 90 + 2(95 + 33 + 17) = \boxed{380}</math>. |
== See also == | == See also == |
Revision as of 14:24, 10 June 2008
Problem
Point is on with and Point is not on so that and and are integers. Let be the sum of all possible perimeters of . Find
Solution
Denote the height of as , , and . Using the Pythagorean theorem, we find that and . Thus, . The LHS is difference of squares, so . As both are integers, must be integral divisors of .
The pairs of divisors of are . This yields the four potential sets for as . The last is not a possibility since it simply degenerates into a line. The sum of the three possible perimeters of is equal to .
See also
2003 AIME I (Problems • Answer Key • Resources) | ||
Preceded by Problem 6 |
Followed by Problem 8 | |
1 • 2 • 3 • 4 • 5 • 6 • 7 • 8 • 9 • 10 • 11 • 12 • 13 • 14 • 15 | ||
All AIME Problems and Solutions |